LSAT Logic Games : Solving grouping games

Study concepts, example questions & explanations for LSAT Logic Games

varsity tutors app store varsity tutors android store

Example Questions

Example Question #21 : Grouping Games

The chef at a La Maison, a popular French restaurant, is famous for making nine different dishes: three beef dishes - A, B and C; three poultry dishes - L, M and N; and three fish dishes - X, Y and Z. La Maison's special tasting menu always consists of exactly five of its chef's famous dishes. Each month, the chef changes the dishes on the tasting menu, choosing from among his famous dishes in accordance with the following conditions:

There must be exactly one fish dish on the menu.

If Z is not on the menu, A cannot be on the menu.

No more than two beef dishes can be on the menu.

If both L and Y are on the menu, B cannot be on the menu.

If A is on the menu, N cannot be on the menu.

Unless both C and X are on the menu, M cannot be on the menu.

For the month of March, which of the following could be the beef and poultry dishes on La Maison's tasting menu?

Possible Answers:

A, B, L, N

C, L, M, N

A, B, L, M

A, B, C, L

B, L, M, N

Correct answer:

C, L, M, N

Explanation:

Each incorrect answer violates at least one of the conditions.

The third condition eliminates {A, B, C, L}. All three beef can't be on the menu.

The fourth condition eliminates {A, B, L, N}. A and N cannot be on the menu together.

The last condition eliminates {A, B, L, M} and {B, L, M, N}. If M is on the menu, C and X must also be on the menu. In each case, M is on the menu without either C or X and with only one dish left to be chosen, which makes fulfillment of the condition impossible.

Example Question #21 : Solving Grouping Games

The chef at a La Maison, a popular French restaurant, is famous for making nine different dishes: three beef dishes - A, B and C; three poultry dishes - L, M and N; and three fish dishes - X, Y and Z. La Maison's special tasting menu always consists of exactly five of its chef's famous dishes. Each month, the chef changes the dishes on the tasting menu, choosing from among his famous dishes in accordance with the following conditions:

There must be exactly one fish dish on the menu.

If Z is not on the menu, A cannot be on the menu.

No more than two beef dishes can be on the menu.

If both L and Y are on the menu, B cannot be on the menu.

If A is on the menu, N cannot be on the menu.

Unless both C and X are on the menu, M cannot be on the menu.

If the chef puts B on June's tasting menu, which of the following pairs of dishes could also be on June's tasting menu?

Possible Answers:

M, N

M, Y

A, X

A, Y

M, Z

Correct answer:

M, N

Explanation:

{A, X} and {A, Y} can be eliminated through a combination of the first and second conditions. If A is on the menu, then Z must be on the menu (second condition). That means that neither X nor Y can also be on the menu (first condition).

{M, Y} and {M, Z} can be eliminated through a combination of the first and last conditions. If M is on the menu, then X must be on the menu (last condition). That means that neither Y nor Z can also be on the menu (first condition).

So, only the correct answer cannot be eliminated.

Example Question #21 : Solving Grouping Games

The chef at a La Maison, a popular French restaurant, is famous for making nine different dishes: three beef dishes - A, B and C; three poultry dishes - L, M and N; and three fish dishes - X, Y and Z. La Maison's special tasting menu always consists of exactly five of its chef's famous dishes. Each month, the chef changes the dishes on the tasting menu, choosing from among his famous dishes in accordance with the following conditions:

There must be exactly one fish dish on the menu.

If Z is not on the menu, A cannot be on the menu.

No more than two beef dishes can be on the menu.

If both L and Y are on the menu, B cannot be on the menu.

If A is on the menu, N cannot be on the menu.

Unless both C and X are on the menu, M cannot be on the menu.

If the chef puts Z on April's tasting menu, which of the following pairs of dishes must also be on April's tasting menu?

Possible Answers:

L, M

L, Y

L, N

A, X

B, M

Correct answer:

L, N

Explanation:

The key to answering this question is recognizing that the first condition (one fish) and the third condition (no more than two beef), taken together with the fact that the tasting menu always consists of five dishes, imply that there must be at least two poultry dishes on any menu.

Then, if Z is on the menu, neither X nor Y can be on the menu (per the first condition). Further, if X cannot be on the menu, neither can M (per the last condition). 

Finally, since there must be at least two poultry dishes on any menu and M cannot be one of those poultry dishes, L and N (the remaining poultry dishes) must both be always be on any menu containing Z.

Example Question #531 : Lsat Logic Games

The chef at a La Maison, a popular French restaurant, is famous for making nine different dishes: three beef dishes - A, B and C; three poultry dishes - L, M and N; and three fish dishes - X, Y and Z. La Maison's special tasting menu always consists of exactly five of its chef's famous dishes. Each month, the chef changes the dishes on the tasting menu, choosing from among his famous dishes in accordance with the following conditions:

There must be exactly one fish dish on the menu.

If Z is not on the menu, A cannot be on the menu.

No more than two beef dishes can be on the menu.

If both L and Y are on the menu, B cannot be on the menu.

If A is on the menu, N cannot be on the menu.

Unless both C and X are on the menu, M cannot be on the menu.

For a given month, which of the following is a list of three dishes that can be chosen together for such month's tasting menu?

Possible Answers:

B, M, N

A, L, X

B, M, Y

A, N, Z

C, M, Y

Correct answer:

B, M, N

Explanation:

The correct answer is the only one that does not violate any of the conditions.

If A is chosen, Z must be chosen (second condition), but then X cannot be chosen (first condition). So, {A, L, X} is eliminated.

A and N cannot be chosen together (fifth condition), so {A, N, Z} is eliminated.

If M is chosen, X must also be chosen (last condition), but then Y cannot also be chosen (first condition). So, {B, M, Y} and {C, M, Y} are eliminated.

Example Question #531 : Lsat Logic Games

Alpha Corporation's Board of Directors (the "Board") consists of exactly five Directors, one of whom is designated as the Chairperson. The Directors are chosen from among a group of five executives of Alpha Corporation - A, B, C, D, and E - and a group of four people not otherwise associated with the company - W, X, Y, and Z. All choices must be made in accordance with the following conditions:

The Board may not include fewer than two Directors from each group.

The Chairperson must be a Director chosen from a group from which exactly one other Director is chosen.

If X is not chosen, A may not be chosen.

If D is not chosen, B may not be chosen.

If either C or E is chosen, the other must also be chosen.

E and W cannot both be chosen.

Which of the following could be a list of the Directors chosen to sit on the Board?

Possible Answers:

A, C, D, W, X

C, D, E, X, Z

C, E, W, X, Z

A, B, X, Y, Z

Correct answer:

C, D, E, X, Z

Explanation:

The correct answer is the only one that does not violate any conditions.

{A, B, X, Y, Z} has B without D, so it violates Condition 4.

{A, C, D, W, X} has C but not E, so it violates Condition 5.

{A, W, X, Y, Z} contains only 1 Director from the first group, so it violates Condition 1.

{C, E, W, X, Z} contains both E and W, so it violates Condition 6.

Example Question #532 : Lsat Logic Games

Each of eight parking lot attendants – F, G, H, J, K, L, N, and O – must be assigned to watch exactly one of three levels – X, Y, and Z - of a multi-level parking facility. Assignments must be made in accordance with the following conditions:

Each level is watched by either two or three of the attendants.

H watches X.

Neither K nor O watches Y.

Neither K nor N watches the same level as J.

If G watches X, both N and O watch Z.

Which one of the following is an acceptable assignment of attendants to parking levels?

Possible Answers:

X: H, J, K

Y: G, N

Z: F, L, O

X: H

Y: F, G, J, L

Z: K, N, O

X: G, H, O

Y: F, J, L

Z: K, N

X: F, G, H

Y: J, L

Z: K, N, O

X: H, N

Y: F, K, L

Z: G, J, O

Correct answer:

X: F, G, H

Y: J, L

Z: K, N, O

Explanation:

All incorrect answers violate at least one condition.

X: H

Y: F, G, J, L

Z: K, N, O

cannot be right because only one attendant is watching X (Condition 1).

 

X: H, N

Y: F, K, L

Z: G, J, O

cannot be right because K is watching Y (Condition 3).

 

X: H, J, K

Y: G, N

Z: F, L, O

cannot be right because J and K are watching the same level, X (Condition 4).

 

X: G, H, O

Y: F, J, L

Z: K, N

cannot be right because G is watching X, while O is not watching Z (Condition 5).

Example Question #27 : Solving Grouping Games

A dance teacher is creating a set list for an upcoming dance recital. There must be five numbers, performed in sequence. The numbers can be solos, duets or trios, made up of eight students: Amy, Belle, Carrie, Dana, Linda, Monique, Nicole and Ona. Each student will perform only once and the set list must conform to the follwing conditions:

 

Amy and Ona must perform solo

Carrie cannot perform in the second number unless Belle performs in the fourth

Nicole and Linda must perform together

Ona must perform at some time before Dana

The first and fifth numbers must be solos

Which of the following is a complete and accurate possible set list?

Possible Answers:

First: Ona; Second: Carrie, Belle; Third: Dana, Nicole, Linda; Fourth: Monique, Dana; Fifth: Amy

First: Amy; Second: Carrie, Nicole, Linda; Third: Ona; Fourth: Dana, Belle; Fifth: Monique

First: Amy; Second: Belle, Dana; Third: Linda, Nicole, Carrie; Fourth: Ona; Fifth: Monique

First: Belle; Second: Monique, Nicole; Third: Ona; Fourth: Dana, Linda, Carrie; Fifth: Amy

First: Ona; Second: Carrie, Amy; Third: Linda, Nicole, Dana; Fourth: Belle; Fifth: Monique

Correct answer:

First: Amy; Second: Carrie, Nicole, Linda; Third: Ona; Fourth: Dana, Belle; Fifth: Monique

Explanation:

This is a simple elimination by rule problem. First we can check for Amy and Ona, eliminating any answers that do not have them performing solo. Then we check for the conditional involving Carrie and Belle - any answer that has Carrie performing second and doesn't have Belle performing fourth is eliminated. Then we can check to make sure all remaining answers have only one performer in the first and last slots (they all do). Then we make sure Nicole and Linda are always performing together. Finally, any leftover answers are eliminated based on the rule that Ona must perform sometime before Dana.

Example Question #22 : Solving Grouping Games

A dance teacher is creating a set list for an upcoming dance recital. There must be five numbers, performed in sequence. The numbers can be solos, duets or trios, made up of eight students: Amy, Belle, Carrie, Dana, Linda, Monique, Nicole and Ona. Each student will perform only once and the set list must conform to the follwing conditions:

 

Amy and Ona must perform solo

Carrie cannot perform in the second number unless Belle performs in the fourth

Nicole and Linda must perform together

Ona must perform at some time before Dana

The first and fifth numbers must be solos

If Carrie performs second and Amy performs first, which of the following could be true?

Possible Answers:

Monique performs third

Ona performs second

Ona performs fifth

Dana performs third

Monique performs second

Correct answer:

Monique performs second

Explanation:

This set up gives us three distinct possibilities. When we place Amy first and Carries second, we automatically add Belle fourth due to the conditional. Since Ona needs to perform alone and also needs to perform before Dana, the only spot left for her is third. Now we get into the possibilities. If Dana performs fourth with Belle, then we place Monique fifth and pair Nicole and Linda with Carrie. This yields the complete set list: First: Amy; Second: Nicole, Linda, Carrie; Third: Ona; Fourth: Dana, Belle; Fifth: Monique. The other option is to place Dana in the fifth number alone. This leaves Nicole, Linda and Monqiue. Since Nicole and Linda must be together, we have the option of placing them either with Carrie second or wth Belle fourth. Monique will be paried with the other. This yields two options, the first being: First: Amy; Second: Carrie, Nicole, Linda; Third: Ona; Fourth: Belle, Monique; Fifth: Dana. The second being: First: Amy; Second: Carrie, Monique; Thrid: Ona; Fourth: Belle, Nicole, Linda; Fifth: Dana. We can look through these possible templates and eliminate any answers that we never see in these hypotheticals.

Example Question #21 : Solving Grouping Games

A dance teacher is creating a set list for an upcoming dance recital. There must be five numbers, performed in sequence. The numbers can be solos, duets or trios, made up of eight students: Amy, Belle, Carrie, Dana, Linda, Monique, Nicole and Ona. Each student will perform only once and the set list must conform to the follwing conditions:

 

Amy and Ona must perform solo

Carrie cannot perform in the second number unless Belle performs in the fourth

Nicole and Linda must perform together

Ona must perform at some time before Dana

The first and fifth numbers must be solos

If Belle performs first, all of the following must be false EXCEPT:

Possible Answers:

Dana performs second

Carrie performs second

Monique performs fifth

Amy performs fifth

Ona performs first

Correct answer:

Amy performs fifth

Explanation:

If Belle performs first we know a few things right away. Since the first performance must be a solo, no one can perform with her. Therefore Ona cannot perform first - the earliest she can perform is second. Since Ona must perform before Dana, Dana cannot perform second. Based on the contrapositive of our conditional, we know that if Belle does not perform fourth, Carrie cannot perform second. And lastly, if Monique performs fifth we know that she must perform by herself. This would mean Amy and Ona would need to be solos in other spots, leaving four dancers to perform at once, which is not possible. 

Example Question #22 : Solving Grouping Games

A dance teacher is creating a set list for an upcoming dance recital. There must be five numbers, performed in sequence. The numbers can be solos, duets or trios, made up of eight students: Amy, Belle, Carrie, Dana, Linda, Monique, Nicole and Ona. Each student will perform only once and the set list must conform to the follwing conditions:

 

Amy and Ona must perform solo

Carrie cannot perform in the second number unless Belle performs in the fourth

Nicole and Linda must perform together

Ona must perform at some time before Dana

The first and fifth numbers must be solos

If Ona performs fourth all of the following could be true EXCEPT:

Possible Answers:

Belle performs third

Monique performs third

Nicole performs second

Carrie performs second

Carrie performs third

Correct answer:

Carrie performs second

Explanation:

If Ona performs second, we know that Dana must perform fifth. Ona must always perform solo, and the fifth spot must always be a solo so these two have to perform alone. Amy must perform solo, so she must go first in the other solo spot. Between spots two and three we must place Carrie, Belle, Nicole, Monique,  and Linda. If Carrie goes second, Belle must go fourth, which we know cannot happen in this scenario. Therefore Carrie must perform third. Nicole and Linda need to perform together, which leaves Belle and Monique to perform together. These two groups can switch between spots two and three. The two distinct possibilities for this scenario are: First: Amy; Second: Nicole, Linda; Third: Carrie, Belle, Monique; Fourth: Ona; Fifth: Dana OR First: Amy; Second: Belle, Monique; Third: Carrie, Nicole, Linda; Fourth: Ona; Fifth: Dana.

Tired of practice problems?

Try live online LSAT prep today.

1-on-1 Tutoring
Live Online Class
1-on-1 + Class
Learning Tools by Varsity Tutors